what number should be added to -5/8 to get -3/2​

Answers

Answer 1

Answer:

-7/8

Step-by-step explanation:

-5/8+x=-3/2

x= -3/2+5/8=-12/8+5/8= -7/8


Related Questions

Help pleaseeeeeeeeeeeeeeeeeeee

Answers

Answer:

{-3, 1, 5, 6}

Step-by-step explanation:

The domain of a relation is the x-values represented in that function. In a coordinate pair, the x-value comes first, so all of the first numbers in each of the pairs are part of the domain. When writing domain, it should always be in the least to greatest order. Therefore, the domain is {-3, 1, 5, 6}. Since this relation is a function, none of the x-values will repeat.

HELP UUUURRRRRRRGGGGGEEEEEENNNNTTTTT PLLLLZZZZZ IM BAD AT MATHHHHHHHH

Answers

Answer:

-1 8/9

Step-by-step explanation:

w + ( - x)

w = -5/9

z = 4/3

Input:

-5/9 + ( -4/3)

-5/9 - 4/3

-4/3 * 3/3 = -12/9

-5/9 - 12/9 = -17/9 = -1 8/9

If my answer is incorrect, pls correct me!

If you like my answer and explanation, mark me as brainliest!

-Chetan K

The area of a triangle is 24 square inches. What is the height of the triangle if the base length is 4 inches?
6 inches
8 inches
12 inches
20 inches

Answers

Answer:

[tex]\boxed {\boxed { \sf 12 \ inches}}[/tex]

Step-by-step explanation:

The area of a triangle can be calculated using the following formula.

[tex]a=\frac{1}{2} bh[/tex]

The area of the triangle is 24 square inches and the base is 4 inches long.

a= 24 in² b= 4 in

Substitute the values into the formula.

[tex]24 \ in^2 = \frac {1}{2} * 4 \ in * h[/tex]

Multiply on the right side of the equation.

[tex]24 \ in ^2 = ( \frac{1}{2} * 4 \ in ) * h[/tex]

[tex]24 \ in ^2 =2 \ in *h[/tex]

We are solving for the height of the triangle, so we must isolate the variable h. It is being multiplied by 2 inches. The inverse of multiplication is division, so we divide both sides by 2 inches.

[tex]\frac { 24 \ in ^2 }{2 \ in }= \frac{ 2 \ in *h}{ 2 \ in}[/tex]

[tex]\frac { 24 \ in ^2 }{2 \ in }= h[/tex]

[tex]12 \ in = h[/tex]

The height of the triangle is 12 inches.

Please help! Determine whether the conjecture is true or false and put an example on why it is

Answers

Answer:

Step-by-step explanation:

The first one is true. There can't be any other choice.

a = 5959599949 b = 0 then a*b = 0 because b = 0

The Second one is also true, although you may stall trying to figure out what is meant.

Suppose the angle to start with is 30 degrees

There are two angles that are supplementary to this angle. They can only be 180 - 30 = 150 each. Therefore they are equal to each other. This happens because supplementary angles must add to 180 and nothing else.

The third one is false. You can think of states like Montana which has 3 syllables and Wyoming which also has 3. Texas has two. But guess what? Maine only has 1.

The last one is also false. If you square an even number, you get an even number. Add 1 and you get an odd number. 4^2 = 16 Add 1 you get 17. Seventeen is odd.

Hi everyone how to solve this question

Answers

$2x+7$

according to the flow chart,

1. multiply [tex] x[/tex] by $2$, so $2x$

and then add seven to it so $2x+7$

note, if the order was reverse, i.e. first add seven ($x+7$). then multiply by two ($2(x+7)$)

the answer would be $2x+14$

Find the area of the parallelogram with vertices A(−1,2,3), B(0,4,6), C(1,1,2), and D(2,3,5).

Answers

Answer:

5*sqrt3

Step-by-step explanation:

The vector AB= (0-(-1), 4-2,6-3) AB= (1,2,3)

The modul of AB is sqrt(1^2+2^2+3^2)= sqrt14

The vector AC is (1-(-1), 1-2, 2-3)= (2,-1,-1)

The modul of B is sqrt (2^2+(-1)^2+(-1)^2)= sqrt6

AB*AC= modul AB*modul AC*cosA

cosA=( 1*2+2*(-1)+3*(-1))/ sqrt14*sqrt6= -3/sqrt84=

sinB= sqrt (1- (-3/sqrt84)^2)= sqrt75/84= sqrt 25/28= 5/sqrt28

s= modul AB*modul AC*sinA= sqrt14*sqrt6* 5/ sqrt28= 5*sqrt3

what should be added to 66.778 get 78.2​

Answers

Answer:

11.422

Step-by-step explanation:

[tex]78.2 - 66.778 \\ = 11.422[/tex]

When csc(Theta)sin(Theta) is simplified, what is the result? StartFraction 1 Over cosecant squared EndFraction StartFraction 1 Over sine squared EndFraction 0 1

Answers

Step-by-step explanation:

csc θ sin θ

(1 / sin θ) sin θ

1

The simplified value of the given expression comes to be 1.

The given expression is:

[tex]cosec\theta.sin\theta[/tex]

What is the trigonometric ratio [tex]cosec\theta[/tex]?

The trigonometric ratio [tex]cosec\theta[/tex] is the ratio of the hypotenuse to the opposite side. It is the inverse of [tex]sin\theta[/tex].

[tex]cosec\theta=\frac{1}{sin\theta}[/tex]

We know that [tex]cosec\theta=\frac{1}{sin\theta}[/tex]

So [tex]cosec\theta.sin\theta[/tex]

[tex]=\frac{1}{sin\theta} .sin\theta[/tex]

=1

So, the simplified value is 1.

Hence, the simplified value of the given expression comes to be 1.

To get more about trigonometric ratios visit:

https://brainly.com/question/24349828

If the errors produced by a forecasting method for 3 observations are +3, +3, and −3, then what is the mean squared error?

Answers

Answer:

9

Step-by-step explanation:

The mean squared error (MSE)of a set of observations can be calculated using the formula :

(1/n)Σ(Actual values - predicted values)^2

Where n = number of observations

Steps :

Error values of each observation (difference between actual and predicted values) is squared.

Step 2:

The squared values are summed

Step 3:

The summation is the divided by the number of observations

The difference between the actual and predicted values is known as the ERROR.

(1/n)Σ(ERROR)^2

n = 3

Error = +3, +3, - 3

MSE = (1/3)Σ[(3)^2 + (3)^2 + (-3)^2]

MSE = (1/3) × [9 + 9 + 9]

MSE = (1/3) × 27

MSE = 9

What is the value of x when y equals 66?

y=0.985897x+0.194185

Answers

Answer:

x = 66.74715005725

Step-by-step explanation:

First you bring over the added variable. 0.194185, and subtract it from 66. Then you divide your difference by 0.985897. This gives you 66.74715005725

Henry takes out a $650 discounted loan with a simple interest rate of 12% for a period of 7 months. How much money does Henry receive into his bank account when the loan is drawn down? Give your answer to the nearest cent.

Answers

Answer:

$546

Step-by-step explanation:

Given

Amount, P = $650

Rate, R = 12%

Period, T = 7 months

Required

Determine the amount paid.

We'll solve this using simple interest formula, as thus

[tex]I = \frac{PRT}{100}[/tex]

Substitute values for T, R and P

[tex]I = \frac{\$650 * 12 * 7}{100}[/tex]

[tex]I = \frac{\$54600}{100}[/tex]

[tex]I = \$546[/tex]

Hence, Henry's withdrawal is $546

solve this equation 4log√x - log 3x =log x^2​

Answers

Answer:

[tex]x = \frac{1}{3} [/tex]

Step-by-step explanation:

*Move terms to the left and set equal to zero:

4㏒(√x) - ㏒(3x) - ㏒(x²) = 0

*simplify each term:

㏒(x²) - ㏒(3x) - ㏒(x²)

㏒(x²÷x²) -㏒(3x)

㏒(x²÷x² / 3x)

*cancel common factor x²:

㏒([tex]\frac{1}{3x}[/tex])

*rewrite to solve for x :

10⁰ = [tex]\frac{1}{3x}[/tex]

1 = [tex]\frac{1}{3x}[/tex]

1 · x = [tex]\frac{1}{3x}[/tex] · x

1x = [tex]\frac{1}{3}[/tex]

*that would be our answer, however, the convention is to exclude the "1" in front of variables so we are left with:

x = [tex]\frac{1}{3}[/tex]


What is the midline equation of the function h(x) = -4 cos(5x - 9) - 7?

Answers

Answer: Midline equation: y = -7

Step-by-step explanation: This function is a sinusoidal function of the form:

y = a.cos(b(x+c))+d

Midline is a horizontal line where the function oscillates above and below.

In the sinusoidal function d represents its vertical shift. Midline is not influenced by any other value except vertical shift. For that reason,

Midline, for the function: [tex]h(x) = -4cos(5x-9) - 7[/tex] is y=d, i.e., [tex]y=-7[/tex]

Answer:

y=-7

Step-by-step explanation:

PLEASE HELP QUICK!!!Suppose the bill for dinner is $16.70, if you want to give a 10% tip what will be the total?

Answers

Answer:

$18.37

Step-by-step explanation:

$16.70 × 1.10 = $18.37

or

$16.70 × 0.10 = $1.67

$16.70 + 1.67 = $18.37

HLP HLP 10 10 10 HLP HLP HLP

W

Answers

Answer:

A. 6²¹

Step-by-step explanation:

When you have a number raised to the power in that form, you have to multiply the powers:

(6⁷)³

7×3 = 21

(6⁷)³ = 6²¹

Answer:

A. 6 raise to 21

Step-by-step explanation:

its a formula if:

a raise to m whole raise to n = a raise to m×n

so here

          6 raise to 7 × 3

         that is 6 to the power of 21

Calculate, correct to one decimal plice
the acute angle between the lines
3x - 4y + 5 = 0 and 2x + 3y -1 = 0
A. 70.69
B. 50.2
C. 39.8
D. 19.4​

Answers

Answer:

A. 70.69 is the correct answer.

Step-by-step explanation:

Given:

Two lines:

[tex]3x - 4y + 5 = 0 \\2x + 3y -1 = 0[/tex]

To find:

Angle between the two lines = ?

Solution:

Acute Angle between two lines can be found by using the below formula:

[tex]tan \theta = |\dfrac{(m_1 - m_2)}{ (1 + m_1m_2)}|[/tex]

Where [tex]\theta[/tex] is the acute angle between two lines.

[tex]m_1, m_2[/tex] are the slopes of two lines.

Slope of a line represented by [tex]ax+by+c=0[/tex] is given as:

[tex]m = -\dfrac{a}{b }[/tex]

So,

[tex]m_1 = -\dfrac{3}{- 4} = \dfrac{3}{4}[/tex]

[tex]m_2 = -\dfrac{2}{ 3}[/tex]

Putting the values in the formula:

[tex]tan \theta = |\dfrac{(\dfrac{3}{4}- (-\dfrac{2}{3}))}{ (1 + \dfrac{3}{4}\times (-\dfrac{2}{3 }))}|\\\Rightarrow tan \theta = |\dfrac{\dfrac{3}{4}+\dfrac{2}{3}}{ (1 -\dfrac{1}{2})}|\\\Rightarrow tan \theta = |\dfrac{\dfrac{17}{12}}{ \dfrac{1}{2}}|\\\Rightarrow tan \theta = \dfrac{17}{6}\\\Rightarrow \theta = tan^{-1}(\frac{17}{6})\\\Rightarrow \theta = \bold{70.69^\circ}[/tex]

So, correct answer is A. 70.69

Solve for h.
H+6/4= 5

Answers

H + 6/4 = 5
- 6/4 -6/4

H = 3.5 OR 3 1/2

What are m and b in the linear equation, using the common meanings of m and b? 2 + 3x + 5 - 2x = y

Answers

y=mx+b is the general formula of linear equation

y=-2x+5+3x+2

y=1x+7

m=1

b=7

Linear equation given in the question is,

2 + 3x + 5 - 2x = y

To simplify this equation further,

Add like terms of the equation,

        (2 + 5) + (3x - 2x) = y

         7 + x = y

Now compare this linear equation with the slope-intercept form of the linear equation,

y = mx + b

Here, m = slope of the line'

b = y-intercept

By comparing the equations,

m = 1

b = 7

Learn more,

https://brainly.com/question/15253236

What would the 60 is x% of 12. Find the value of x.

Answers

Answer:

The value of x= 20

Step-by-step explanation:

I believe the question is ,"60% of x is us, find x"

So , if the percentage of x to 60 is 12.

60/100 * x = 12

0.6 *x = 12

Dividing both sides by 0.6

X= 12/0.6

X= (12/6) *(10)

X= 2*10

.x= 20

The value of x= 20

I have this question on an assignment and my calculator won't show the horizontal asymptote correctly can I get some help here?

Answers

What's the question? I can try and help..

(Small sample confidence intervals for a population mean) suppose you are taking a sampling of 15 measurements. you find that x=75 and s =5. assuming normality, the 99% confidence interval for the population mean is:__________

Answers

Answer:

The 99% confidence interval is  [tex]71.67 < \mu < 78.33[/tex]

Step-by-step explanation:

From the question we are told that

     The sample  size  is  [tex]n = 15[/tex]

      The  sample  mean is  [tex]\= x = 75[/tex]

        The  standard deviation is  [tex]s = 5[/tex]

 Given that confidence is  99%  then the level of significance is mathematically represented as

              [tex]\alpha = 100 - 99[/tex]

             [tex]\alpha = 1\%[/tex]

             [tex]\alpha = 0.01[/tex]

Next we obtain the critical values of  [tex]\frac{ \alpha }{2}[/tex] from the normal distribution table

   The  value is

                  [tex]Z_{\frac{ \alpha }{2} } = 2.58[/tex]

Generally the margin for error is mathematically represented as

            [tex]E = Z_{\frac{ \alpha }{2} } * \frac{ s}{ \sqrt{n} }[/tex]

=>         [tex]E = 2.58 * \frac{ 5}{ \sqrt{15} }[/tex]

=>         [tex]E = 3.3307[/tex]

   The  99% confidence interval is mathematically represented as

             [tex]\= x -E < \mu < \= x +E[/tex]

=>          [tex]75 - 3.3307 < \mu <75 + 3.3307[/tex]

=>          [tex]71.67 < \mu < 78.33[/tex]

Select the correct answer from each drop-down menu.
The function f is given by the table of values as shown below.

x 1 2 3 4 5
f(x) 13 19 37 91 253
Use the given table to complete the statements.

The parent function of the function represented in the table is
.

If function f was translated down 4 units, the
-values would be
.

A point in the table for the transformed function would be
.

Answers

Answer:

3^x9, 15, 33, 87, 249(4, 87) for example

Step-by-step explanation:

a) First differences of the f(x) values in the table are ...

  19 -13 = 6, 37 -19 = 18, 91 -37 = 54, 253 -91 = 162

The second differences are not constant:

  18 -6 = 12, 54 -18 = 36, 162 -54 = 108

But, we notice that both the first and second differences have a common ratio. This is characteristic of an exponential function. The common ratio is 18/6 = 3, so the parent function is 3^x.

__

b) Translating a function down 4 units subtracts 4 from each y-value. The values of f(x) in the table would be ...

  9, 15, 33, 87, 249

__

c) The x-values of the function stay the same for a vertical translation, so the points in the table of the transformed function are ...

  (x, f(x)) = (1, 9), (2, 15), (3, 33), (4, 87), (5, 249)

Answer: I think this is it:

The parent function of the function represented in the table is exponential. If function f was translated down 4 units, the f(x)-values would be decreased by 4. A point in the table for the transformed function would be (4,87)

Step-by-step explanation: I got it right on Edmentum!

Given there are 26 alphabets in the English language, how many possible three-letter words are there?

Answers

Answer: 17,576

We have 26 letters and 3 slots to fill. We can reuse a letter if it has been picked, so we have 26^3 = 26*26*26 = 17,576 different three letter "words". I put that in quotes because a lot of the words aren't actual words, but more just a sequence of letters.

a mens clothing sore sold out of $50 jackets and $30 jackets for a total of $2360 if the store sold 12 more$30 jackets than$50 jackets how many$50 jackets were sold

Answers

Answer:

25

Step-by-step explanation:

Let x represent the number of $50 jackets that were sold, and let y represent how many $30 jackets were sold.

50x + 30y = 2360

y = x + 12

Solve by substitution by substituting the second equation into the first one. Then, solve for x:

50x + 30y = 2360

50x + 30(x + 12) = 2360

50x + 30x + 360 = 2360

80x + 360 = 2360

80x = 2000

x = 25

So, 25 $50 jackets were sold.

find the equation of the line that is perpendicular to y=6x-2) and contains to the point (6-,2)

Answers

Answer:

y = -1/6x - 1.

Step-by-step explanation:

I  am assuming that the point id (6, -2).

The slope of the required line = -1/6.

y - y1 = m(x - x1) where m = slope and x1,y1 is a point on the line so we have

y - (-2) = -1/6( x- 6)

y + 2 = -1/6x + 1

y = -1/6x - 1.

Pamela is 8 years older that Jiri. The sum of their age is 102. What is Jiri's age?

Answers

Answer:

Step-by-step explanation:

102-8= 94

94/2= 47

Jiri is 47 years old.

trigonometric identities

Answers

Without knowing what Juan's exact steps were, it's hard to say what he did wrong. The least you could say is that his solution is simply not correct.

4 sin²(θ) - 1 = 0

==>   sin²(θ) = 1/4

==>   sin(θ) = ±1/√2

==>   θ = π/4, 3π/4, 5π/4, 7π/4

divide 111001 by 1101​

Answers

Based on the fact that you asked this three times and got the same answer three times, I suspect the interpretation made by the users that posted those answers was incorrect, and that you meant to ask about dividing in base 2.

We have

111001₂ = 1×2⁵ + 1×2⁴ + 1×2³ + 1×2⁰ = 57

1101₂ = 1×2³ + 1×2² + 1×2⁰ = 13

and 57/13 = (4×13 + 5)/13 = 4 + 5/13.

4 = 2² is already a power of 2, so we have

111001₂/1101₂ = 1×2² + 5/13

we just need to convert 5/13. To do this, we look for consecutive negative powers of 2 that 5/13 falls between, then expand 5/13 as the sum of the smaller power of 2 and some remainder term. For instance,

• 1/4 < 5/13 < 1/2, and

5/13 - 1/4 = (20 - 13)/52= 7/52

so that

5/13 = 1/4 + 7/52

or

5/13 = 1×2 ⁻² + 7/52

Then a partial conversion into base 2 gives us

111001₂/1101₂ = 1×2² + 1×2 ⁻² + 7/52

111001₂/1101₂ = 100.01₂ + 7/52

Continuing in this fashion, we find

• 1/8 < 7/52 < 1/4, and

7/52 = 1/8 + 1/104

==>   111001₂/1101₂ = 100.011₂ + 1/104

• 1/128 < 1/104 < 1/64, and

1/104 = 1/128 + 3/1664

==>   111001₂/1101₂ = 100.0110001₂ + 3/1664

• 1/1024 < 3/1664 < 1/512, and

3/1664 = 1/1024 + 11/13312

==>   111001₂/1101₂ = 100.0110001001₂ + 11/13312

• 1/2048 < 11/13312 < 1/1024, and

11/13312 = 1/2048 + 9/26624

==>   111001₂/1101₂ = 100.01100010011₂ + 9/26624

• 1/4096 < 9/26624 < 1/2048, and

9/26624 = 1/4096 + 5/53248

==>   111001₂/1101₂ = 100.011000100111₂ + 5/53248

and so on.

It turns out that this pattern repeats, so that

[tex]\displaystyle \frac{111001_2}{1101_2} = 100.\overline{011000100111}_2[/tex]

Matrix A is said to be involutory if A2 = I. Prove that a square matrix A is both orthogonal and involutory if and only if A is symmetric.

Answers

Answer:

4 · 1/4 (I-0) = (A-0)∧2

see details in the graph

Step-by-step explanation:

Matrix A is expressed in the form A∧2=I

To proof that Matrix A is both orthogonal and involutory, if and only if A is symmetric is shown by re-expressing that

A∧2=I in the standard form

4 · 1/4 (I-0) = (A-0)∧2

Re-expressing

A∧2 = I as a graphical element plotted on the graph

X∧2=I

The orthogonality is shown in the graphical plot displayed in the picture. Orthogonality expresses the mutually independent form of two vectors expressed in their perpendicularity.

Find the volume of the following figure round your answer to the nearest tenth and if necessary use pi

Answers

Answer:

1526.04

Step-by-step explanation:

the formula for calculating the volume of cone is

V=πr^2(h/3)

Thus,

V = (3.14)(9)^2(18/3)

V = (3.14)(81)(6)

V = 1536.04 yd^3

Rounding off to the nearest tenth, we get

V = 1536 yd^3

Other Questions
How many numbers between 1 and 1000 do not contain the digits 8, and 9 ? Mrs. Casanello had a breast biopsy last Friday. She calls Wednesday afternoon and asks if the results are back yet. The results came in Wednesday morning, and you know that the results were good: there is no malignancy. What should you tell Mrs. Casanello? Solving Equations by Dividing 2) 9x= -135 Solve for x. 0 -144 O 126 O 15 0 -15 Which expression is equivalent to the area of metal sheet required to make this square-shaped traffic sign? A sign labeled 2x-1 on one side. A: 4x^2 - 1 B:4x^2 + 1 C: 4x^2 + 4x - 1 D: 4x^2 - 4x + 1 how was human society organized? describe in brief Explain why Pope Urban II helped Byzantine at the start of the Crusades. Quick Company's lease payments are made at the end of each period. Quick's liability for a capital lease will be reduced periodically by the minimum lease payment, adjusted by: please help to do it read this excerpt dear sir or madam i am responding to your ad seeking data managers why is this cover letter inapporiate? HELPPLSFind all the missing elements: Suppose that Y1, Y2,..., Yn denote a random sample of size n from a Poisson distribution with mean . Consider 1 = (Y1 + Y2)/2 and 2 = Y . Derive the efficiency of 1 relative to 2. Find the solution of the system of equations.2x 10y = -28-10x + 10y = -20GbA Which graph represents a function? Write an expression in simplified form for the area of each rectangle. Width: 5+2y Length:5 The Government Accounting Office (GAO) announces deep cuts to social security, Medicare, and welfare programs. Which determinant of aggregate demand causes the change Please give me the answer ASAP The average of 5 numbers is 7. If one of the five numbers is removed, the average of the four remaining numbers is 6. What is the value of the number that was removed Show Your Work Which of the following is the term for tightly packed sheets of cells that cover organs and outer surfaces?a. Epithelial tissueb. Connective tissuec. Muscle tissued. Nervous tissue In complete sentences, explain the following: Is 700 a good estimate for the sum of 435 and 368? If it is, explain why it is a good estimate. If it is not, explain why it is a bad estimate. Yooo I just had MAD diarrhea D: Suppose you buy 6 cans of peaches at $1.10 each, 5 cans of corn for $.89 each, and 3 boxes of breakfast cereal at $3.52 each. a. Write three expressions; one each that shows how to determine the total spent on peaches,corn and breakfast cereal. (3 pts)